Google Groups no longer supports new Usenet posts or subscriptions. Historical content remains viewable.
Dismiss

Help with question about equivalence principle

0 views
Skip to first unread message

Duwayne Anderson

unread,
Mar 31, 2003, 2:22:43 PM3/31/03
to
Dear readers:

An engineer that I know claims to have an argument that the
equivalence principle is not valid/accurate. If any of you feel
energetic enough to help me answer his question, would you email me so
that I can send you a copy of his short 2-page argument?

Here it is, in text:

Figure 1 shows a box. The box can be in a uniform gravitational
field, or in constant acceleration. This represents a laboratory with
two identical Clocks. At t = to Clocks A and B are synchronized. At t
= t1 Clock B is raised to level 2 and left there for a while. At t =
t2 Clock B is lowered back to level 1 and the difference between
Clocks A and B is recorded.

Task: Determine what influence is causing the force as presented in
Figure 1. Explain how to detect each type of influence.

Given: The force required to keep every object separated in, Figure 1,
at a constant distance from the floor is caused by one of
the following:

1 Uniform Gravitational Field – Magnitude and direction of Force are
independent of position in laboratory.
2 Linear acceleration – Magnitude and direction of Force are
independent of position in laboratory.
3 Circular motion (centripetal acceleration). – Magnitude and
direction of Force are dependent on position in
laboratory. This is by default as there is no other possibility.

Note: It is assumed that if the force is caused by acceleration, the
velocities involved are significantly less than the speed of light.

Based on concepts from Einstein's Theories, it is understood that:
• If the force is caused by Gravity, then the natural frequency of
Clock B should increase if it is moved from level 1 to 2.
• If the force is caused by Linear acceleration, then after Clock B is
moved to level 2, it will continue to be accelerated and move with the
same velocity as Clock A.

Therefore, the natural frequency of Clock B at level 2 will be the
same as the natural frequency of Clock A at level 1. However, the
transition process of moving Clock B from level 1 to 2 will cause a
slight shift in the indicated time of Clock B. This is because during
the transition process it will not be accelerated at the same rate or
moving with the same velocity as Clock A.

However, this influence should be very small compared to the overall
results if Clock B is left at level 2 for a sufficient amount of time.
( Note: This approach neglects the secondary or minor influence of
length contraction on the laboratory).

• If the force is caused by Circular motion, then moving Clock B form
level 1 to 2 should cause a reduction in the velocity of Clock B with
a corresponding increase in natural frequency. The influence of motion
parallel to the floor (+x, -x, +z, -z directions) on natural frequency
and perceived force will also be asymmetric.

Approach – Conduct the following experiment
1 At t = to synchronized Clocks A and B
2 At t = t1 move Clock B from level 1 to level 2 and wait for a while
3 At t = t2 move Clock B from level 2 back to level 1 and record the
indicated time of each Clock.
Check difference between Clocks A and B
If the indicated time of Clock B ≤≤≤≤Clock A:
Then the force is caused by linear acceleration
Note: The "less than or equal to" logic is used to account for the
transient process of moving Clock B from level 1 to 2 and back again.
Done
If the indicated time of Clock B > Clock A
Then the force is caused by gravity or centripetal acceleration and
another test is required.
Test influence of motion parallel to the floor (+x, -x, +z, -z
directions) on natural frequency or perceived force.
If the influence of motion is symmetric
Then the force is caused by gravity
Else
If the influence of motion is asymmetric
Then the force is caused by centripetal acceleration

Thanks in advance for any comments/suggestions you might have.

My email is duwayne.r...@tek.com

Duwayne Anderson

Bilge

unread,
Mar 31, 2003, 2:41:04 PM3/31/03
to
Duwayne Anderson:
>Dear readers:
>
>An engineer that I know claims to have an argument that the
>equivalence principle is not valid/accurate. If any of you feel
>energetic enough to help me answer his question, would you email me so
>that I can send you a copy of his short 2-page argument?

Looks like a homework assignment.

Hayek

unread,
Mar 31, 2003, 2:50:00 PM3/31/03
to

Duwayne Anderson wrote:

> Dear readers:
>
> An engineer that I know claims to have an argument that the
> equivalence principle is not valid/accurate. If any of you feel
> energetic enough to help me answer his question, would you email me so
> that I can send you a copy of his short 2-page argument?
>
> Here it is, in text:
>
> Figure 1 shows a box. The box can be in a uniform gravitational
> field, or in constant acceleration. This represents a laboratory with
> two identical Clocks. At t = to Clocks A and B are synchronized. At t
> = t1 Clock B is raised to level 2 and left there for a while. At t =
> t2 Clock B is lowered back to level 1 and the difference between
> Clocks A and B is recorded.
>
> Task: Determine what influence is causing the force as presented in
> Figure 1. Explain how to detect each type of influence.
>
> Given: The force required to keep every object separated in, Figure 1,
> at a constant distance from the floor is caused by one of
> the following:
>

> 1 Uniform Gravitational Field - Magnitude and direction of Force are


> independent of position in laboratory.

> 2 Linear acceleration - Magnitude and direction of Force are


> independent of position in laboratory.

> 3 Circular motion (centripetal acceleration). - Magnitude and


> direction of Force are dependent on position in
> laboratory. This is by default as there is no other possibility.
>
> Note: It is assumed that if the force is caused by acceleration, the
> velocities involved are significantly less than the speed of light.
>
> Based on concepts from Einstein's Theories, it is understood that:

> . If the force is caused by Gravity, then the natural frequency of


> Clock B should increase if it is moved from level 1 to 2.

> . If the force is caused by Linear acceleration, then after Clock B is


> moved to level 2, it will continue to be accelerated and move with the
> same velocity as Clock A.
>
> Therefore, the natural frequency of Clock B at level 2 will be the
> same as the natural frequency of Clock A at level 1. However, the
> transition process of moving Clock B from level 1 to 2 will cause a
> slight shift in the indicated time of Clock B. This is because during
> the transition process it will not be accelerated at the same rate or
> moving with the same velocity as Clock A.
>
> However, this influence should be very small compared to the overall
> results if Clock B is left at level 2 for a sufficient amount of time.
> ( Note: This approach neglects the secondary or minor influence of
> length contraction on the laboratory).
>

> . If the force is caused by Circular motion, then moving Clock B form


> level 1 to 2 should cause a reduction in the velocity of Clock B with
> a corresponding increase in natural frequency. The influence of motion
> parallel to the floor (+x, -x, +z, -z directions) on natural frequency
> and perceived force will also be asymmetric.
>

> Approach - Conduct the following experiment


> 1 At t = to synchronized Clocks A and B
> 2 At t = t1 move Clock B from level 1 to level 2 and wait for a while
> 3 At t = t2 move Clock B from level 2 back to level 1 and record the
> indicated time of each Clock.
> Check difference between Clocks A and B
> If the indicated time of Clock B ≤≤≤≤Clock A:
> Then the force is caused by linear acceleration
> Note: The "less than or equal to" logic is used to account for the
> transient process of moving Clock B from level 1 to 2 and back again.
> Done
> If the indicated time of Clock B > Clock A
> Then the force is caused by gravity or centripetal acceleration and
> another test is required.
> Test influence of motion parallel to the floor (+x, -x, +z, -z
> directions) on natural frequency or perceived force.
> If the influence of motion is symmetric
> Then the force is caused by gravity
> Else
> If the influence of motion is asymmetric
> Then the force is caused by centripetal acceleration
>
> Thanks in advance for any comments/suggestions you might have.
>
> My email is duwayne.r...@tek.com

Slap on my head.
Why did'nt I think of that.
It was so simple.

I think he is absolutely right.

Hayek.

--
The small particles wave at
the big stars and get noticed.
:-)

Duwayne Anderson

unread,
Mar 31, 2003, 6:59:59 PM3/31/03
to
dub...@radioactivex.lebesque-al.net (Bilge) wrote in message news:<slrnb8haet....@radioactivex.lebesque-al.net>...

No, it's not "homework." I think I know the answer, and have an
argument that I've made (by the way, I don't think the equivalency
principle is violated) but I'd like to get additional points of view
on this.

Are there any serious scientists on this newsgroup who could wade in
with an explanation?

Duwayne Anderson

Alan McIntire

unread,
Mar 31, 2003, 10:09:44 PM3/31/03
to
duwa...@hotmail.com (Duwayne Anderson) wrote in message news:<a42139e3.03033...@posting.google.com>...

> Dear readers:
>
> An engineer that I know claims to have an argument that the
> equivalence principle is not valid/accurate. If any of you feel
> energetic enough to help me answer his question, would you email me so
> that I can send you a copy of his short 2-page argument?
>
> Here it is, in text:
>
> Figure 1 shows a box. The box can be in a uniform gravitational
> field, or in constant acceleration. This represents a laboratory with
> two identical Clocks. At t = to Clocks A and B are synchronized. At t
> = t1 Clock B is raised to level 2 and left there for a while. At t =
> t2 Clock B is lowered back to level 1 and the difference between
> Clocks A and B is recorded.
>
> Task: Determine what influence is causing the force as presented in
> Figure 1. Explain how to detect each type of influence.
>
> Given: The force required to keep every object separated in, Figure 1,
> at a constant distance from the floor is caused by one of
> the following:
>
> 1 Uniform Gravitational Field ? Magnitude and direction of Force are

> independent of position in laboratory.
> 2 Linear acceleration ? Magnitude and direction of Force are

> independent of position in laboratory.
> 3 Circular motion (centripetal acceleration). ? Magnitude and

> direction of Force are dependent on position in
> laboratory. This is by default as there is no other possibility.
>
> Note: It is assumed that if the force is caused by acceleration, the
> velocities involved are significantly less than the speed of light.
>
> Based on concepts from Einstein's Theories, it is understood that:
> ? If the force is caused by Gravity, then the natural frequency of

> Clock B should increase if it is moved from level 1 to 2.
> ? If the force is caused by Linear acceleration, then after Clock B is

> moved to level 2, it will continue to be accelerated and move with the
> same velocity as Clock A.
>
> Therefore, the natural frequency of Clock B at level 2 will be the
> same as the natural frequency of Clock A at level 1. However, the
> transition process of moving Clock B from level 1 to 2 will cause a
> slight shift in the indicated time of Clock B. This is because during
> the transition process it will not be accelerated at the same rate or
> moving with the same velocity as Clock A.
>
> However, this influence should be very small compared to the overall
> results if Clock B is left at level 2 for a sufficient amount of time.
> ( Note: This approach neglects the secondary or minor influence of
> length contraction on the laboratory).
>
> ? If the force is caused by Circular motion, then moving Clock B form

> level 1 to 2 should cause a reduction in the velocity of Clock B with
> a corresponding increase in natural frequency. The influence of motion
> parallel to the floor (+x, -x, +z, -z directions) on natural frequency
> and perceived force will also be asymmetric.
>
> Approach ? Conduct the following experiment

> 1 At t = to synchronized Clocks A and B
> 2 At t = t1 move Clock B from level 1 to level 2 and wait for a while
> 3 At t = t2 move Clock B from level 2 back to level 1 and record the
> indicated time of each Clock.
> Check difference between Clocks A and B
> If the indicated time of Clock B &#8804;&#8804;&#8804;&#8804;Clock A:
> Then the force is caused by linear acceleration
> Note: The "less than or equal to" logic is used to account for the
> transient process of moving Clock B from level 1 to 2 and back again.
> Done
> If the indicated time of Clock B > Clock A
> Then the force is caused by gravity or centripetal acceleration and
> another test is required.
> Test influence of motion parallel to the floor (+x, -x, +z, -z
> directions) on natural frequency or perceived force.
> If the influence of motion is symmetric
> Then the force is caused by gravity
> Else
> If the influence of motion is asymmetric
> Then the force is caused by centripetal acceleration
>
> Thanks in advance for any comments/suggestions you might have.
>
> My email is duwayne.r...@tek.com
>
> Duwayne Anderson

There are two factors at work here. As you said, let the time taken
to move clock B from level 1 to level 2 and back again be negligible
compared to the total time spent at level 2.

Gravitational potential will make clock B at level two be faster by a
factor of
1/SQRT (1 - (2GM/r*c*c))
and slower because of increased speed around the rotating earth by a
factor of
( 1 - (V/c)*2)/(1-(w/c)^2)
Where V is the velocity around the earth of the clock at B, which is
faster than the velocity, w, of the clock at A around the earth.
Combine the two parts of the equation to get the net speeding up or
slowing down of clock B

Jeff Krimmel

unread,
Mar 31, 2003, 11:19:26 PM3/31/03
to
"Duwayne Anderson" <duwa...@hotmail.com> wrote in message
news:a42139e3.03033...@posting.google.com...

[snip]

> Are there any serious scientists on this newsgroup who could wade in
> with an explanation?

Try posting at sci.physics.research if you are looking for expert opinion.

Jeff


and...@attglobal.net

unread,
Apr 1, 2003, 4:22:32 AM4/1/03
to
Duwayne Anderson wrote:
>
> Dear readers:
>
> An engineer that I know claims to have an argument that the
> equivalence principle is not valid/accurate.

That engineer wouldn't be you would it?

John Anderson

Paul B. Andersen

unread,
Apr 1, 2003, 8:51:49 AM4/1/03
to

"Duwayne Anderson" <duwa...@hotmail.com> skrev i melding
news:a42139e3.03033...@posting.google.com...

> Dear readers:
>
> An engineer that I know claims to have an argument that the
> equivalence principle is not valid/accurate. If any of you feel
> energetic enough to help me answer his question, would you email me so
> that I can send you a copy of his short 2-page argument?
>
> Here it is, in text:
>
> Figure 1 shows a box. The box can be in a uniform gravitational
> field, or in constant acceleration. This represents a laboratory with
> two identical Clocks. At t = to Clocks A and B are synchronized. At t
> = t1 Clock B is raised to level 2 and left there for a while. At t =
> t2 Clock B is lowered back to level 1 and the difference between
> Clocks A and B is recorded.
>
> Task: Determine what influence is causing the force as presented in
> Figure 1. Explain how to detect each type of influence.
>
> Given: The force required to keep every object separated in, Figure 1,
> at a constant distance from the floor is caused by one of
> the following:
>
> 1 Uniform Gravitational Field - Magnitude and direction of Force are

> independent of position in laboratory.
> 2 Linear acceleration - Magnitude and direction of Force are

> independent of position in laboratory.
> 3 Circular motion (centripetal acceleration). - Magnitude and

> direction of Force are dependent on position in
> laboratory. This is by default as there is no other possibility.

You can exclude case 3.
Your frame of reference is accelerated and rotating.
You can of course detect that it is rotating.

> Note: It is assumed that if the force is caused by acceleration, the
> velocities involved are significantly less than the speed of light.
>
> Based on concepts from Einstein's Theories, it is understood that:

> . If the force is caused by Gravity, then the natural frequency of


> Clock B should increase if it is moved from level 1 to 2.

As observed from level 1, yes.

> . If the force is caused by Linear acceleration, then after Clock B is


> moved to level 2, it will continue to be accelerated and move with the
> same velocity as Clock A.
>
> Therefore, the natural frequency of Clock B at level 2 will be the
> same as the natural frequency of Clock A at level 1.

Wrong.
The clocks will behave in the same way in either case.

The rest is thus wrong.

Paul


Duwayne Anderson

unread,
Apr 1, 2003, 10:23:11 AM4/1/03
to
and...@attglobal.net wrote in message news:<3E895A...@attglobal.net>...

No (although I am a Principle Engineer with Tektronix). The person
I'm having this discussion with is a relative who thinks the principle
of equivalency is incorrect.

I've found a couple of sites that describe the "rocket" problem (both
associated with universities) and both argue that the clock at the top
of the rocket runs faster than the one at the base -- in accordance
with the principle of equivalency. One such article is at

http://www.pa.uky.edu/~cvj/as500_lec6/as500_lec6.html

and the other article is at

http://www.phys.ufl.edu/~acosta/phy3101/lectures/relativity5.pdf

The arguments seem pretty solid, but my relative remains unconvinced
(though he has provided no argument that the clocks run the same --
just his asserted opinion that they do).

So, do you have some "light" to shed on this?

Duwayne Anderson

Duwayne Anderson

unread,
Apr 1, 2003, 10:24:32 AM4/1/03
to
mcin...@earthlink.net (Alan McIntire) wrote in message news:<48658b64.0303...@posting.google.com>...
<snip>

> There are two factors at work here. As you said, let the time taken
> to move clock B from level 1 to level 2 and back again be negligible
> compared to the total time spent at level 2.
>
> Gravitational potential will make clock B at level two be faster by a
> factor of
> 1/SQRT (1 - (2GM/r*c*c))

There is no gravitational potential involved. The rocket is not in a
gravitational field. It is simply accelerating at a constant rate of
acceleration.

<snip to end>

Duwayne Anderson

Duwayne Anderson

unread,
Apr 1, 2003, 10:28:37 AM4/1/03
to
Hayek <hay...@nospam.xs4all.nl> wrote in message news:<3E889BE...@nospam.xs4all.nl>...

<snip>


> Slap on my head.
> Why did'nt I think of that.
> It was so simple.
>
> I think he is absolutely right.

Would you please comment on your above opinion with regard to the
articles at

http://www.phys.ufl.edu/~acosta/phy3101/lectures/relativity5.pdf

and at

http://www.pa.uky.edu/~cvj/as500_lec6/as500_lec6.html

Both seem to argue convincingly that the clock at the top of the
rocket will run faster than the one at the bottom when the clocks are
in a rocket that is uniformly accelerating, but not in a gravitational
field.

It's an experimental fact that clocks at different heights in a
gravitational field run at different rates, so the example of clocks
in the rocket would seem to be fully consistent with the principle of
equivalency. This conclusion is in opposition, however, to the person
making the argument, as he believes the principle of equivalency is
incorrect.

Duwayne Anderson

Duwayne Anderson

unread,
Apr 1, 2003, 10:29:46 AM4/1/03
to
"Jeff Krimmel" <madscie...@hotmail.com> wrote in message news:<ir8ia.20682$C86.1...@twister.austin.rr.com>...

Ah, Jeff. Thanks. I was beginning to be depressed by the low
signal-to-noise level here.

Best,

Duwayne Anderson

Joe Fischer

unread,
Mar 31, 2003, 11:57:50 PM3/31/03
to
Duwayne Anderson <duwa...@hotmail.com> wrote:
: An engineer that I know claims to have an argument that the

: equivalence principle is not valid/accurate.

Please explain to him that there are many different
impressions of what the equivalence principle, or more
importantly, what "Einstein's Principle of Equivalence"
means.

Newton's formula is derived from postulating
the equivalence of inertial mass and gravitational mass.

Einstein's Principle of Equivalence is the basis
of the concepts of General Relativity, but it originally
only treated an upward accelerating surface.
The mature theory treats the problem in four
dimensions, so that the original 2-D concept only
applies to a very small region.

: If any of you feel


: energetic enough to help me answer his question, would you email me so
: that I can send you a copy of his short 2-page argument?

Thanks anyway, but there are more arguments and
opinions than anybody wants to read.
General Relativity is supported by every experiment
ever done, that outweighs any opinion or argument.

Joe Fischer

--
3

Duwayne Anderson

unread,
Apr 1, 2003, 1:27:38 PM4/1/03
to
"Paul B. Andersen" <paul.b....@hia.no> wrote in message news:<b6c5hl$5hk$1...@dolly.uninett.no>...

Paul, thanks for your comments.

Duwayne

Duwayne Anderson

unread,
Apr 1, 2003, 1:43:44 PM4/1/03
to
Joe Fischer <grav...@shell1.iglou.com> wrote in message news:<3e891...@news.iglou.com>...

> Duwayne Anderson <duwa...@hotmail.com> wrote:
> : An engineer that I know claims to have an argument that the
> : equivalence principle is not valid/accurate.
>
> Please explain to him that there are many different
> impressions of what the equivalence principle, or more
> importantly, what "Einstein's Principle of Equivalence"
> means.
>
> Newton's formula is derived from postulating
> the equivalence of inertial mass and gravitational mass.

Understood. In my explanations, I pointed out (as is done by several
authors) that this equivalence exists in Newtonian physics.

> Einstein's Principle of Equivalence is the basis
> of the concepts of General Relativity, but it originally
> only treated an upward accelerating surface.
> The mature theory treats the problem in four
> dimensions, so that the original 2-D concept only
> applies to a very small region.

Agreed.

>
> : If any of you feel
> : energetic enough to help me answer his question, would you email me so
> : that I can send you a copy of his short 2-page argument?
>
> Thanks anyway, but there are more arguments and
> opinions than anybody wants to read.
> General Relativity is supported by every experiment
> ever done, that outweighs any opinion or argument.

Agreed. Please understand that I'm having a debate with someone who,
for philosophical reasons (I think) rejects the idea of equivalence
between a gravitational field and acceleration.

When he first broached the issue he used the standard arguments about
non-parallel lines of force in a gravitational field around the earth
to persuade me that the equivalence principle is not correct. When
that failed (I pointed out that the principle of equivalence can be
reduced to a statement about uniform fields) he's moved to essentially
postulating an experiment and asserting the experimental results as an
argument that the equivalence principle is not correct. He's become
somewhat agitated with my arguments showing that the clock at the top
of the rocket really does run at a different rate than the one at the
bottom, and has finally retreated to the "science is mostly wrong"
haven of religious fanatics.

Still, this is someone that I'm close to, and I'd like some means of
helping him understand the equivalence principle in this case. I was
hoping that there might be someone on this newsgroup who could explain
it better than I.

For example I have (and if I'm wrong, please correct me) tried to
explain it this way:

Suppose we use for our clock in the top of the rocket a frequency
stabilized laser. We use a partially transmissive mirror to let some
of the light out of the laser. We split this output light so that
some goes to the observer in the top of the rocket, and some goes to
the observer at the bottom of the rocket. The observer at the bottom
of the rocket has an identical clock.

We tell "time" with out clock by counting cycles. So we begin, and
the rocket accelerates. The clock at the top has a specific frequency
with respect to the observer there. And the clock at the bottom has a
specific frequency with respect to the observer there. But the light
from the top clock, which reaches the observer at the bottom, is blue
shifted. You can make the argument for this blue shift several
different ways, but it's hard to make the point with classical
arguments (because the reasons are not classical!).

At any rate, with the light from the top clock being blue shifted when
observed at the bottom, the result is that the person at the bottom
observes the clock at the top running faster. This is exactly what
happens with a gravitational field, and so the principle of
equivalence is validated.

So, two questions:

1) Have I explained this correctly? I think I have, and have found
several references produced by universities that support this
argument.

2) Is there a *better* way to explain this to someone who is
questioning the principle of equivalence in the first place?

Best regards,

Duwayne Anderson

Joe Fischer

unread,
Apr 1, 2003, 6:51:38 PM4/1/03
to
Duwayne Anderson <duwa...@hotmail.com> wrote:
: Joe Fischer wrote:
:> General Relativity is supported by every experiment

:> ever done, that outweighs any opinion or argument.
:
: Agreed. Please understand that I'm having a debate with someone who,
: for philosophical reasons (I think) rejects the idea of equivalence
: between a gravitational field and acceleration.

The concept of a real physical entity called
"a gravitational field" is very popular, simply because
Newtonian concepts dominate gravitational physics.
This is true for two reasons, Newtonian gravitation
is the first learned theory, like a first language, and
that makes it hard to dismiss.
Also, Newtonian gravitation is simple to work with,
and that makes it popular.

But it is not as precise as General Relativity,
and it is completely wrong in the basic concepts.

: When he first broached the issue he used the standard arguments about


: non-parallel lines of force in a gravitational field around the earth
: to persuade me that the equivalence principle is not correct. When
: that failed (I pointed out that the principle of equivalence can be
: reduced to a statement about uniform fields) he's moved to essentially
: postulating an experiment and asserting the experimental results as an
: argument that the equivalence principle is not correct.

New experiments are needed, but putting them in
practice is very difficult, because the measurement
difference between Newtonian gravitation and General
Relativity are extremely small in ordinary circumstances.

: He's become


: somewhat agitated with my arguments showing that the clock at the top
: of the rocket really does run at a different rate than the one at the
: bottom,

All perfect clocks run at the same rate, regardless
of where they are.
This is a confusing situation, and needs to be
studied at a high level to understand it.

: and has finally retreated to the "science is mostly wrong"
: haven of religious fanatics.

Einstein's Principle of Equivalence is more
of an important working tool than a "principle".
For instance, the "horizon" of a black hole
is often considered to be accelerating outward at
almost the speed of light, and thinking about it
that way permits making conclusions that are
correct.

: Still, this is someone that I'm close to, and I'd like some means of


: helping him understand the equivalence principle in this case. I was
: hoping that there might be someone on this newsgroup who could explain
: it better than I.

I don't call it the "equivalence principle"
because it seems to me that most people that use
that wording are simply stating that inertial mass
is equal to gravitational mass, rather than my
belief that they are two terms for the same quantity.
I think "Principle of Equivalence" specifies
more clearly that Einstein did propose an important
working tool as a 'principle", rather than the less
useful Newtonian premise that gravitational mass
is a separate attribute of matter that is simply
equal to inertial mass.

: For example I have (and if I'm wrong, please correct me) tried to


: explain it this way:
:
: Suppose we use for our clock in the top of the rocket a frequency
: stabilized laser. We use a partially transmissive mirror to let some
: of the light out of the laser. We split this output light so that
: some goes to the observer in the top of the rocket, and some goes to
: the observer at the bottom of the rocket. The observer at the bottom
: of the rocket has an identical clock.
:
: We tell "time" with out clock by counting cycles. So we begin, and
: the rocket accelerates. The clock at the top has a specific frequency
: with respect to the observer there. And the clock at the bottom has a
: specific frequency with respect to the observer there. But the light
: from the top clock, which reaches the observer at the bottom, is blue
: shifted. You can make the argument for this blue shift several
: different ways, but it's hard to make the point with classical
: arguments (because the reasons are not classical!).

I think that the reasons are classical, only
the problem is usually done using Euclidean space and
constant dimensions.

: At any rate, with the light from the top clock being blue shifted when


: observed at the bottom, the result is that the person at the bottom
: observes the clock at the top running faster. This is exactly what
: happens with a gravitational field, and so the principle of
: equivalence is validated.

He sees a higher frequency, not a faster clock.

: So, two questions:


:
: 1) Have I explained this correctly? I think I have, and have found
: several references produced by universities that support this
: argument.

Try to get the raw data explanation of the
"tower experiment", it makes clear that frequency
changes are to be expected.

: 2) Is there a *better* way to explain this to someone who is


: questioning the principle of equivalence in the first place?

: Duwayne Anderson

I think learning the terminology of General Relativity,
like "proper time", "coordinate acceleration", etc. will
eventually make clear that observations are biased by
any observer.
This terminology of GR is very different from
Newtonian terminology.
Trying to discuss gravity theory without specifying
which theory each term references is confusing.

Joe Fischer

--
3

and...@attglobal.net

unread,
Apr 2, 2003, 12:52:21 AM4/2/03
to
Gravitational redshift and its relationship to the principle of
equivalence is explained very well in Weinberg, Gravitation and
Cosmology. You need to define an experiment to compare something
with the two clock rates. If they exchange a light signal, the
frequency of the light signal is constant wrt coordinate time
in an appropriate metric in a stationary spacetime (like the earth's).
The two clocks measure proper time at their locations which is related
to coordinate time by

ds = sqrt(g_0_0)*dt

where s is proper time, t is coordinate time and g_0_0 is the time
time coefficient of the metric.

And this experiment was done by Pound and Rebka and the GR analysis
agrees with the result.

John Anderson

Duwayne Anderson

unread,
Apr 1, 2003, 11:09:17 PM4/1/03
to
Joe Fischer <grav...@shell1.iglou.com> wrote in message news:<3e8a2...@news.iglou.com>...

> Duwayne Anderson <duwa...@hotmail.com> wrote:
> : Joe Fischer wrote:
> :> General Relativity is supported by every experiment
> :> ever done, that outweighs any opinion or argument.
> :
> : Agreed. Please understand that I'm having a debate with someone who,
> : for philosophical reasons (I think) rejects the idea of equivalence
> : between a gravitational field and acceleration.
>
> The concept of a real physical entity called
> "a gravitational field" is very popular, simply because
> Newtonian concepts dominate gravitational physics.
> This is true for two reasons, Newtonian gravitation
> is the first learned theory, like a first language, and
> that makes it hard to dismiss.
> Also, Newtonian gravitation is simple to work with,
> and that makes it popular.

I agree.

>
> But it is not as precise as General Relativity,
> and it is completely wrong in the basic concepts.

Again, I agree. The issue with this individual with whom I'm having
the conversation is that he says clocks in a gravitational "field"
will behave differently from clocks in an accelerating reference
frame. Yet GR says they will behave the same.

Right?


>
> : When he first broached the issue he used the standard arguments about
> : non-parallel lines of force in a gravitational field around the earth
> : to persuade me that the equivalence principle is not correct. When
> : that failed (I pointed out that the principle of equivalence can be
> : reduced to a statement about uniform fields) he's moved to essentially
> : postulating an experiment and asserting the experimental results as an
> : argument that the equivalence principle is not correct.
>
> New experiments are needed, but putting them in
> practice is very difficult, because the measurement
> difference between Newtonian gravitation and General
> Relativity are extremely small in ordinary circumstances.

Actually, it seems to me that the experimental data for this example
may already exist in linear accelerators. Muons have a relatively
short life (microseconds) in a stationary reference frame, but a much
longer life in a relativistic reference frame. Since huge
accelerations are used in linear accelerators, it seems that GR must
be used to fully describe the measured halflife of muons in those
machines. This would be a direct test for this individual's proposed
experiment, I think.

>
> : He's become
> : somewhat agitated with my arguments showing that the clock at the top
> : of the rocket really does run at a different rate than the one at the
> : bottom,
>
> All perfect clocks run at the same rate, regardless
> of where they are.

In the local frame of reference. But not all clocks agree on the time
between events.

> This is a confusing situation, and needs to be
> studied at a high level to understand it.

I don't think time dilation is an issue here. My friend accepts that,
and it is an experimental fact.

>
> : and has finally retreated to the "science is mostly wrong"
> : haven of religious fanatics.
>
> Einstein's Principle of Equivalence is more
> of an important working tool than a "principle".
> For instance, the "horizon" of a black hole
> is often considered to be accelerating outward at
> almost the speed of light, and thinking about it
> that way permits making conclusions that are
> correct.

I agree. But remember, my friend does not accept this. He believes
that gravitational "fields" and acceleration are not the same -- that
you can devise an experiment that can tell the difference between the
two (the difference between linear acceleration and a uniform
gravitational "field.")

> : Still, this is someone that I'm close to, and I'd like some means of
> : helping him understand the equivalence principle in this case. I was
> : hoping that there might be someone on this newsgroup who could explain
> : it better than I.
>
> I don't call it the "equivalence principle"
> because it seems to me that most people that use
> that wording are simply stating that inertial mass
> is equal to gravitational mass, rather than my
> belief that they are two terms for the same quantity.
> I think "Principle of Equivalence" specifies
> more clearly that Einstein did propose an important
> working tool as a 'principle", rather than the less
> useful Newtonian premise that gravitational mass
> is a separate attribute of matter that is simply
> equal to inertial mass.

Don't acceleration and gravitational fields (I say fields because GR
is, after all, a classical theory) treat time dilation the same?

>
> : For example I have (and if I'm wrong, please correct me) tried to
> : explain it this way:
> :
> : Suppose we use for our clock in the top of the rocket a frequency
> : stabilized laser. We use a partially transmissive mirror to let some
> : of the light out of the laser. We split this output light so that
> : some goes to the observer in the top of the rocket, and some goes to
> : the observer at the bottom of the rocket. The observer at the bottom
> : of the rocket has an identical clock.
> :
> : We tell "time" with out clock by counting cycles. So we begin, and
> : the rocket accelerates. The clock at the top has a specific frequency
> : with respect to the observer there. And the clock at the bottom has a
> : specific frequency with respect to the observer there. But the light
> : from the top clock, which reaches the observer at the bottom, is blue
> : shifted. You can make the argument for this blue shift several
> : different ways, but it's hard to make the point with classical
> : arguments (because the reasons are not classical!).
>
> I think that the reasons are classical, only
> the problem is usually done using Euclidean space and
> constant dimensions.

I don't understand what you mean by "the reasons are classical." They
are certainly not Newtonian.

> : At any rate, with the light from the top clock being blue shifted when
> : observed at the bottom, the result is that the person at the bottom
> : observes the clock at the top running faster. This is exactly what
> : happens with a gravitational field, and so the principle of
> : equivalence is validated.
>
> He sees a higher frequency, not a faster clock.

Not true. The clock rate is determined by the frequency. They are
one and the same.

> : So, two questions:
> :
> : 1) Have I explained this correctly? I think I have, and have found
> : several references produced by universities that support this
> : argument.
>
> Try to get the raw data explanation of the
> "tower experiment", it makes clear that frequency
> changes are to be expected.

Yes. But that is in a gravitational field. My friend accepts that,
but thinks the result would be different if conducted in an
accelerating rocket.

>
> : 2) Is there a *better* way to explain this to someone who is
> : questioning the principle of equivalence in the first place?
> : Duwayne Anderson
>
> I think learning the terminology of General Relativity,
> like "proper time", "coordinate acceleration", etc. will
> eventually make clear that observations are biased by
> any observer.

If by that you mean that observations depend on reference frame, I
agree.

> This terminology of GR is very different from
> Newtonian terminology.
> Trying to discuss gravity theory without specifying
> which theory each term references is confusing.
>
> Joe Fischer


True. But again, the issue is with someone who does not accept one of
the key operational principles of GR.

Duwayne Anderson

eshal

unread,
Apr 1, 2003, 11:45:36 PM4/1/03
to

"Duwayne Anderson" <duwa...@hotmail.com> wrote in message
news:a42139e3.03040...@posting.google.com...

Duwayne... you've come to the right place. Now, don't just stand around like
that.

Mate, you have walked into something of a battle zone and this NG is one of
the fronts currently engaged in heavy exchanges.

Care to join? lol....

The Equivalence is gaining ground. (-:

em


Hayek

unread,
Apr 2, 2003, 12:41:21 AM4/2/03
to

Duwayne Anderson wrote:

> Hayek <hay...@nospam.xs4all.nl> wrote in message
> news:<3E889BE...@nospam.xs4all.nl>...
>
> <snip>
>
>> Slap on my head. Why did'nt I think of that. It
>> was so simple.
>>
>> I think he is absolutely right.
>>
>
> Would you please comment on your above opinion with
> regard to the articles at
>
>
http://www.phys.ufl.edu/~acosta/phy3101/lectures/relativity5.pdf
>

>
> and at
>
> http://www.pa.uky.edu/~cvj/as500_lec6/as500_lec6.html
>
> Both seem to argue convincingly that the clock at
> the top of the rocket will run faster than the one
> at the bottom when the clocks are in a rocket that
> is uniformly accelerating, but not in a
> gravitational field.
>
> It's an experimental fact that clocks at different
> heights in a gravitational field run at different
> rates, so the example of clocks in the rocket would
> seem to be fully consistent with the principle of
equivalency.
>


I think the Equivalence principle is *not* even
applicable here. Einstein specifically stipulated that
the laws of physics must be the same for every *local*
observer. I am working on a theory, or a viewpoint on
Relativity that explains it mechanically. The
gravitational field is also an inertial field. GR
defines it that way. With the local observer, is
actually meant, in the mechanical explanation, the
observer in a flat inertial field.

Now this observer, in case (1) cheats. He does a
non-local experiment. Since he actually measures the
gradient of his inertial field.

In case(2), there should be no gradient. Because on the
first floor of the rocket, the acceleration should be
the same. If you measure the g-force on the first floor,
it is the same as the g-force on the rocket's floor,
contrary to the situation on Earth. In the rocket, it is
a *local* experiment.

But I find this experiment is very interesting. Thanks
for bringing it up. Could you let me know if you make
progress on it ? I will also do my best in clearing
things up.


This conclusion is in opposition, however, to the person
> making the argument, as he believes the principle of
> equivalency is incorrect.


There are many interpretations of the principle.
I am almost there in giving a complete mechanical
explanation for Relativity. This experiment is a nice
litmus test. If hope my explanation does not fail here,
or I am back to square 1.

Duwayne Anderson

unread,
Apr 2, 2003, 10:06:15 AM4/2/03
to
and...@attglobal.net wrote in message news:<3E8A7A...@attglobal.net>...

John:

Thanks for your comments. You say the experiment was done by Pound
and Rebka. Can you provide a citation? Is the experimental result
found in Weinberg?

Duwayne Anderson

Joe Fischer

unread,
Apr 2, 2003, 4:04:41 PM4/2/03
to
Duwayne Anderson <duwa...@hotmail.com> wrote:
: Joe Fischer wrote:
:> Also, Newtonian gravitation is simple to work with,

:> and that makes it popular.
:
: I agree.
:
:> But it is not as precise as General Relativity,
:> and it is completely wrong in the basic concepts.
:
: Again, I agree. The issue with this individual with whom I'm having
: the conversation is that he says clocks in a gravitational "field"
: will behave differently from clocks in an accelerating reference
: frame. Yet GR says they will behave the same.
:
: Right?

You seem to make assumptions about General Relativity
that may or may not be true, the very wording can make a
big difference.

Physics everywhere the same means all perfect
clocks keep perfect time.
The facts about this are blurred by experiments
that try to compare clocks moving relative to each other
using radio (photon) signals, and that is difficult or
even an invalid endeavor.

There is no evidence whatsoever for a physical
entity "gravitational field" except how free-moving
objects move.

:> : When he first broached the issue he used the standard arguments about


:> : non-parallel lines of force in a gravitational field around the earth
:> : to persuade me that the equivalence principle is not correct. When
:> : that failed (I pointed out that the principle of equivalence can be
:> : reduced to a statement about uniform fields) he's moved to essentially
:> : postulating an experiment and asserting the experimental results as an
:> : argument that the equivalence principle is not correct.
:>
:> New experiments are needed, but putting them in
:> practice is very difficult, because the measurement
:> difference between Newtonian gravitation and General
:> Relativity are extremely small in ordinary circumstances.
:
: Actually, it seems to me that the experimental data for this example
: may already exist in linear accelerators. Muons have a relatively
: short life (microseconds) in a stationary reference frame, but a much
: longer life in a relativistic reference frame.

I don't think so. I think they just move further
than ordinary math predicts when they are moving fast, but
acceleration is not part of the reason.

: Since huge


: accelerations are used in linear accelerators, it seems that GR must
: be used to fully describe the measured halflife of muons in those
: machines. This would be a direct test for this individual's proposed
: experiment, I think.

I think you are making wrong assumptions about
half-life, lifetimes, and what is measured.

:> : He's become


:> : somewhat agitated with my arguments showing that the clock at the top
:> : of the rocket really does run at a different rate than the one at the
:> : bottom,
:>
:> All perfect clocks run at the same rate, regardless
:> of where they are.
:
: In the local frame of reference. But not all clocks agree on the time
: between events.

All perfect clocks next to each other agree on
the time between events, relativity is about relative
motion. The clock in a GPS satellite is built to
run slower than the receiver clock to compensate for
the increased frequency that is caused by the elevator
floor accelerating upward, to follow the principle of
equivalence to logical conclusions, predictions and
understanding, it is necessary to remain in Einstein's
elevator.
There is also the factor of relative velocity
to be considered and built into the GPS clock rate,
but that is a complex issue.

:> This is a confusing situation, and needs to be


:> studied at a high level to understand it.
:
: I don't think time dilation is an issue here. My friend accepts that,
: and it is an experimental fact.

I don't think time dilation occurs, it is
an observation phenomenon.

:> : and has finally retreated to the "science is mostly wrong"


:> : haven of religious fanatics.
:>
:> Einstein's Principle of Equivalence is more
:> of an important working tool than a "principle".
:> For instance, the "horizon" of a black hole
:> is often considered to be accelerating outward at
:> almost the speed of light, and thinking about it
:> that way permits making conclusions that are
:> correct.
:
: I agree. But remember, my friend does not accept this. He believes
: that gravitational "fields" and acceleration are not the same -- that
: you can devise an experiment that can tell the difference between the
: two (the difference between linear acceleration and a uniform
: gravitational "field.")

I suggest that you just consider him to be an
unwavering Newtonian and let it go at that.
Casual conversations and news releases by most
scientists are in Newtonian terms, so your friend is
in the majority.
It is very difficult to tell how many people
really understand General Relativity and there are
very few people who major in General Relativity.

:> : Still, this is someone that I'm close to, and I'd like some means of


:> : helping him understand the equivalence principle in this case. I was
:> : hoping that there might be someone on this newsgroup who could explain
:> : it better than I.
:>
:> I don't call it the "equivalence principle"
:> because it seems to me that most people that use
:> that wording are simply stating that inertial mass
:> is equal to gravitational mass, rather than my
:> belief that they are two terms for the same quantity.
:> I think "Principle of Equivalence" specifies
:> more clearly that Einstein did propose an important
:> working tool as a 'principle", rather than the less
:> useful Newtonian premise that gravitational mass
:> is a separate attribute of matter that is simply
:> equal to inertial mass.
:
: Don't acceleration and gravitational fields (I say fields because GR
: is, after all, a classical theory) treat time dilation the same?

No, obviously there is an apparent gradient in
the observed changes of motion of free-moving objects
near large bodies of matter.

So it appears your assumption about the Principle
of Equivalence is too idealistic.
The PoE is a tool and a clue to the proper way
to think about gravity, something more is needed.

You can read about my speculations on the matter
by searching

www.groups.google.com

for

divergent matter

and trace the author trail.

:> : For example I have (and if I'm wrong, please correct me) tried to


:> : explain it this way:
:> :
:> : Suppose we use for our clock in the top of the rocket a frequency
:> : stabilized laser. We use a partially transmissive mirror to let some
:> : of the light out of the laser. We split this output light so that
:> : some goes to the observer in the top of the rocket, and some goes to
:> : the observer at the bottom of the rocket. The observer at the bottom
:> : of the rocket has an identical clock.
:> :
:> : We tell "time" with out clock by counting cycles. So we begin, and
:> : the rocket accelerates. The clock at the top has a specific frequency
:> : with respect to the observer there. And the clock at the bottom has a
:> : specific frequency with respect to the observer there. But the light
:> : from the top clock, which reaches the observer at the bottom, is blue
:> : shifted. You can make the argument for this blue shift several
:> : different ways, but it's hard to make the point with classical
:> : arguments (because the reasons are not classical!).
:>
:> I think that the reasons are classical, only
:> the problem is usually done using Euclidean space and
:> constant dimensions.
:
: I don't understand what you mean by "the reasons are classical." They
: are certainly not Newtonian.

No, but Newtonian is a model concept, there is
a more basic nature concept. Space may be considered
as flat because space is not a physical entity.
There are many concepts that have to be thought
of as basic and fundamental, and there is certainly a
physical causal mechanism of gravitation, it is more
than just "how you think about it".

:> : At any rate, with the light from the top clock being blue shifted when


:> : observed at the bottom, the result is that the person at the bottom
:> : observes the clock at the top running faster. This is exactly what
:> : happens with a gravitational field, and so the principle of
:> : equivalence is validated.
:>
:> He sees a higher frequency, not a faster clock.
:
: Not true. The clock rate is determined by the frequency. They are
: one and the same.

No, you are not keeping track of emitted frequency
and received frequency between objects that may be moving
relative to each other even though the distance between
them appears constant.

In the tower experiment, the same results would
be obtained in the elevator if the length of the tower
was constantly lengthening (in addition to the acceleration
of the elevator).

:> : So, two questions:


:> :
:> : 1) Have I explained this correctly? I think I have, and have found
:> : several references produced by universities that support this
:> : argument.
:>
:> Try to get the raw data explanation of the
:> "tower experiment", it makes clear that frequency
:> changes are to be expected.
:
: Yes. But that is in a gravitational field. My friend accepts that,
: but thinks the result would be different if conducted in an
: accelerating rocket.

The result would be different, the issues here
are that you want the PoE to be idealistic and work in
all cases, but gravitational fields can have different
configurations or shapes.
The accelerating elevator creates a uniform
gravitational field, planets create a gradient field
in most cases. The gravitational field of the
Earth does not have a consistent gradient, outside
the surface it is consistent, but below the surface
it can diminish less than commonly thought, and it
even increases with depth at some point, contrary
to what a homogeneous sphere would produce.

:> : 2) Is there a *better* way to explain this to someone who is


:> : questioning the principle of equivalence in the first place?
:> : Duwayne Anderson
:>
:> I think learning the terminology of General Relativity,
:> like "proper time", "coordinate acceleration", etc. will
:> eventually make clear that observations are biased by
:> any observer.
:
: If by that you mean that observations depend on reference frame, I
: agree.

Not "on reference frame", but on relative motion,
and the elementary relative motion may not be what the
observer thinks it is.
That is the essence of Einstein's first musings
about a train on a rail next to another train, the
observer has difficulty telling which train is moving.

Joe Fischer

--
3

and...@attglobal.net

unread,
Apr 3, 2003, 12:35:06 AM4/3/03
to

Phys. Rev. Letters 4, 337 (1960)

If you don't know how to find a citation, you shouldn't be asking
for one. This is a classical experiment that supports GR.

> Is the experimental result
> found in Weinberg?
>

Yes.

John Anderson

Duwayne Anderson

unread,
Apr 3, 2003, 10:17:12 AM4/3/03
to
and...@attglobal.net wrote in message news:<3E8BC8...@attglobal.net>...

Thanks.

<snip rest of prattle>

Duwayne Anderson

Duwayne Anderson

unread,
Apr 3, 2003, 10:30:13 AM4/3/03
to
Joe Fischer <grav...@shell1.iglou.com> wrote in message news:<3e8b5...@news.iglou.com>...

<snip>


> Physics everywhere the same means all perfect
> clocks keep perfect time.

No, that's not what it means at all. There is nothing sacred about
time that makes it invariant.

> The facts about this are blurred by experiments
> that try to compare clocks moving relative to each other
> using radio (photon) signals, and that is difficult or
> even an invalid endeavor.

It's an experimental fact that clocks measure different amounts of
time between events. For example, place an atomic clock on the
ground, and another one in an airplane. Now have the airplane take
off and fly around the world. When it lands, compare the clocks.
They will have different readings. The difference between the
readings is greater than can be attributed to clock error.

> There is no evidence whatsoever for a physical
> entity "gravitational field" except how free-moving
> objects move.

That word "except" is rather interesting.

Look. Rather than debate semantics with you, I'd like to couch the
issue in terms of experiments done, and results measured.

Experiment #1: Suppose I take two atomic clocks. I leave one on the
ground. I raise the other to 10,000 feet and then return it to the
ground, next to the first clock. I compare their readings. They will
be different.

Experiment #2: Now suppose that I take the same two atomic clocks
from experiment #1, and put them in a rocket. I blast my rocket out
into space, far away from any suns or planets. Next, I accelerate my
rocket at the constant rate of 9.8 m/s. While my rocket is
accelerating, I take one of my clocks and move it to the nose cone
10,000 feet away (this is a really long rocket). Then I bring it back
to the base of the rocket, and compare the readings on the clocks.
Will the clocks read the same, or will their reading be different, as
in experiment #1?

My friend asserts that the readings on the clocks will be the same in
experiment #2, while I say they will be different, as in experiment
#1.

Duwayne Anderson

unread,
Apr 3, 2003, 12:29:54 PM4/3/03
to
and...@attglobal.net wrote in message news:<3E8BC8...@attglobal.net>...

John:

Thanks for your help and posting the citation. I don't have access to
a library with the Phys. Rev. Letters going back to 1960. But I have
familiarized myself with that experiment and its results from
second-hand sources.

While interesting, and confirming GR., the paper does not address
directly the issue that my friend has in mind.

See, he accepts the gravitational red shift. That's what the P&R
experiment was about. What he does *not* accept is the idea that two
clocks in an accelerating rocket ship will disagree. He thinks they
will agree; I say they won't necessarily (depending on the
experiment).

Here's the thought experiment he proposes:

1) Take a tall rocket. Park it on earth. Start with two identical
clocks at the bottom of the rocket. Move one of the clocks to the top
of the rocket, and then back down. They will disagree in their
readings -- in accordance with the P&R experiment.

2) Take the same rocket and two clocks and move them far out into
space, away from any gravitating bodies. Accelerate the rocket at a
constant 9.8 m/sec^2. Starting with both clocks at the rocket's base,
move one clock to the top of the rocket, and then back down. I claim
the readings will disagree as they did in the first experiment, while
my friend claims they will not disagree. He says they will have the
same reading.

He proposes test #2 as a way of telling the difference between
accelerating in space away from any massive bodies, and sitting at
relative rest on a planet like earth.

Now, the P&R experiment validates test #1, but I don't think it says
anything of test #2. What I'm looking for is actual experimental
verification of test #2. I understand the mathematical reasoning
behind claims that the clocks will read differently, but that's not
acceptable to my friend. He wants to see experimental verification.
Do you know of any?

Best regards,

Duwayne Anderson

Lawrence Foard

unread,
Apr 3, 2003, 1:22:05 PM4/3/03
to
In article <a42139e3.03040...@posting.google.com>,

Duwayne Anderson <duwa...@hotmail.com> wrote:
>
>Thanks for your help and posting the citation. I don't have access to
>a library with the Phys. Rev. Letters going back to 1960. But I have
>familiarized myself with that experiment and its results from
>second-hand sources.
>
>While interesting, and confirming GR., the paper does not address
>directly the issue that my friend has in mind.
>
>See, he accepts the gravitational red shift. That's what the P&R
>experiment was about. What he does *not* accept is the idea that two
>clocks in an accelerating rocket ship will disagree. He thinks they
>will agree; I say they won't necessarily (depending on the
>experiment).

Actually red shift arguments should be sufficient to find that they don't
agree. A laser is pointed upward from the bottom of the ship. An observer
higher in the ship will observe a red shift. This do to the fact that the
ship is moving faster when the light arrives at the detector than when it
left the laser. Because the distance between them is not changing, it can
only consistently be viewed (atleast in there accelerated reference frame)
as there clocks running at different rates.

>Here's the thought experiment he proposes:
>
>1) Take a tall rocket. Park it on earth. Start with two identical
>clocks at the bottom of the rocket. Move one of the clocks to the top
>of the rocket, and then back down. They will disagree in their
>readings -- in accordance with the P&R experiment.
>
>2) Take the same rocket and two clocks and move them far out into
>space, away from any gravitating bodies. Accelerate the rocket at a
>constant 9.8 m/sec^2. Starting with both clocks at the rocket's base,
>move one clock to the top of the rocket, and then back down. I claim
>the readings will disagree as they did in the first experiment, while
>my friend claims they will not disagree. He says they will have the
>same reading.

[...]

>He wants to see experimental verification.
>Do you know of any?

This would be pretty hard to do, you'd need a really big rocket engine and a
very very sensitive clock. However the red shift between the bottom and top
is something that obviously must happen when the system is viewed by
an inertial observer. I don't know of any other way to describe this in the
frame of the rocket other than as clocks running slower on the bottom.
--
Be a counter terrorist perpetrate random senseless acts of kindness
Rave: Immanentization of the Eschaton in a Temporary Autonomous Zone.
"Anyone who trades liberty for security deserves neither liberty nor security"
-Benjamin Franklin

Joe Fischer

unread,
Apr 4, 2003, 5:28:20 AM4/4/03
to
Duwayne Anderson <duwa...@hotmail.com> wrote:
: Joe Fischer wrote:
:> Physics everywhere the same means all perfect

:> clocks keep perfect time.
:
: No, that's not what it means at all. There is nothing sacred about
: time that makes it invariant.

I didn't say time was invariant, but there is
nothing that will make an ideal clock speed up or slow down.
Rate of time flow is tied to the length of measuring
rods, as is the speed of light.

:> The facts about this are blurred by experiments


:> that try to compare clocks moving relative to each other
:> using radio (photon) signals, and that is difficult or
:> even an invalid endeavor.
:
: It's an experimental fact that clocks measure different amounts of
: time between events. For example, place an atomic clock on the
: ground, and another one in an airplane. Now have the airplane take
: off and fly around the world. When it lands, compare the clocks.
: They will have different readings. The difference between the
: readings is greater than can be attributed to clock error.

But if this were true, it would be documented
with numerous experiments, especially with the Space
Shuttle, and long term experiments in aircraft, rather
than a single flight or trip.

Note that early experiments were flawed due
to clocks that were affected by the shock of landing.

:> There is no evidence whatsoever for a physical


:> entity "gravitational field" except how free-moving
:> objects move.
:
: That word "except" is rather interesting.
:
: Look. Rather than debate semantics with you, I'd like to couch the
: issue in terms of experiments done, and results measured.

Fine, but real experiments, over long periods,
with definitive results.
Wouldn't your premise require the affect to
be additive over long periods?

: Experiment #1: Suppose I take two atomic clocks. I leave one on the


: ground. I raise the other to 10,000 feet and then return it to the
: ground, next to the first clock. I compare their readings. They will
: be different.

That isn't an experiment, it is a plan and opinionated
prediction.

: Experiment #2: Now suppose that I take the same two atomic clocks


: from experiment #1, and put them in a rocket. I blast my rocket out
: into space, far away from any suns or planets. Next, I accelerate my
: rocket at the constant rate of 9.8 m/s. While my rocket is
: accelerating, I take one of my clocks and move it to the nose cone
: 10,000 feet away (this is a really long rocket). Then I bring it back
: to the base of the rocket, and compare the readings on the clocks.
: Will the clocks read the same, or will their reading be different, as
: in experiment #1?

My opinion is that if both clock are accelerated
equally in any direction, they will always read the same.
I think atomic clocks might be affected by any
acceleration which might affect the emission of atoms.

: My friend asserts that the readings on the clocks will be the same in


: experiment #2, while I say they will be different, as in experiment
: #1.

: Duwayne Anderson

Maybe you can get Ken Seto to help explain
how velocity causes a change in rate of time flow,
and how orbiting at 3000 miles can cause a change
in rate of time flow.

I only maintain that any consistent and
repeatable experiment could have been done many
times on aircraft and the Space Shuttle without
depending on a few isolated experiments.

Joe Fischer

--
3

Duwayne Anderson

unread,
Apr 4, 2003, 10:30:51 AM4/4/03
to
Joe Fischer <grav...@shell1.iglou.com> wrote in message news:<3e8d5...@news.iglou.com>...

> Duwayne Anderson <duwa...@hotmail.com> wrote:
> : Joe Fischer wrote:
> :> Physics everywhere the same means all perfect
> :> clocks keep perfect time.
> :
> : No, that's not what it means at all. There is nothing sacred about
> : time that makes it invariant.
>
> I didn't say time was invariant, but there is
> nothing that will make an ideal clock speed up or slow down.

When compared to each other in different reference frames they do.
It's an experimental fact.

> Rate of time flow is tied to the length of measuring
> rods, as is the speed of light.
>
> :> The facts about this are blurred by experiments
> :> that try to compare clocks moving relative to each other
> :> using radio (photon) signals, and that is difficult or
> :> even an invalid endeavor.
> :
> : It's an experimental fact that clocks measure different amounts of
> : time between events. For example, place an atomic clock on the
> : ground, and another one in an airplane. Now have the airplane take
> : off and fly around the world. When it lands, compare the clocks.
> : They will have different readings. The difference between the
> : readings is greater than can be attributed to clock error.
>
> But if this were true, it would be documented
> with numerous experiments,

There are.

During October 1971, a remarkable experiment was performed that
checked both gravitational red shift and time dilation, by measuring
directly their effects on traveling clocks. The actual experiment was
coordinated by J. C. Hfele, then of the Washington University in St.
Louis, and Richard Keating of the U.S. Naval Observatory, using
cesium-beam atomic clocks. They flew their clocks on commercial
aircraft during regularly scheduled flights (and because of government
restrictions, could not even fly first class). The clocks were
positioned against the front wall of the coach-class cabin to protect
them from sudden motions. The eastward trip took place between
October 4 and 7 and included 41 hours in flight, while the westward
trip took place between October 13 and 17, and included 49 hours of
flight. For the westward flight the predicted gain in the flying
clock was 275 nanoseconds, of which two-thirds was due to
gravitational blue shift; the observed gain was 273 nanoseconds. For
the eastward flight, the time dilation was predicted to give a loss
larger than the gain due to the gravitational blue shift, the net
being a loss of 40 nanoseconds; the observed loss was 59 nanoseconds.
These results agreed exactly with General Relativity, to within the
experimental error of +/- 20 nanoseconds. The experimental error was
attributed to inaccuracies in the flight data and intrinsic variations
in the rates of cesium clocks.

Similar experiments were conducted by Bob Vessot and NASA in 1976 when
they flew an atomic clock in low earth orbit and then compared it's
time (after the clock was brought back to earth) with that of an
identical clock kept at the NASA tracking station at Merritt Island.
When the clocks were brought back together their relative time had
shifted in exact agreement with the General Theory of Relativity. [See
"Was Einstein Right? Putting General Relativity to the Test," by
Clifford M. Will, Basic Books, 1986, pages 42-43; 54-57.]

<snip to end>

Duwayne Anderson

American Quarter Horse: The ultimate all-terrain vehicle.

Duwayne Anderson

unread,
Apr 4, 2003, 10:34:34 AM4/4/03
to
ent...@farviolet.com (Lawrence Foard) wrote in message news:<b6hu4d$tvb$1...@farviolet.com>...

> In article <a42139e3.03040...@posting.google.com>,
> Duwayne Anderson <duwa...@hotmail.com> wrote:
> >
> >Thanks for your help and posting the citation. I don't have access to
> >a library with the Phys. Rev. Letters going back to 1960. But I have
> >familiarized myself with that experiment and its results from
> >second-hand sources.
> >
> >While interesting, and confirming GR., the paper does not address
> >directly the issue that my friend has in mind.
> >
> >See, he accepts the gravitational red shift. That's what the P&R
> >experiment was about. What he does *not* accept is the idea that two
> >clocks in an accelerating rocket ship will disagree. He thinks they
> >will agree; I say they won't necessarily (depending on the
> >experiment).
>
> Actually red shift arguments should be sufficient to find that they don't
> agree.

True. I agree, but my friend is not convinced.

> A laser is pointed upward from the bottom of the ship. An observer
> higher in the ship will observe a red shift. This do to the fact that the
> ship is moving faster when the light arrives at the detector than when it
> left the laser. Because the distance between them is not changing, it can
> only consistently be viewed (atleast in there accelerated reference frame)
> as there clocks running at different rates.
>
> >Here's the thought experiment he proposes:
> >
> >1) Take a tall rocket. Park it on earth. Start with two identical
> >clocks at the bottom of the rocket. Move one of the clocks to the top
> >of the rocket, and then back down. They will disagree in their
> >readings -- in accordance with the P&R experiment.
> >
> >2) Take the same rocket and two clocks and move them far out into
> >space, away from any gravitating bodies. Accelerate the rocket at a
> >constant 9.8 m/sec^2. Starting with both clocks at the rocket's base,
> >move one clock to the top of the rocket, and then back down. I claim
> >the readings will disagree as they did in the first experiment, while
> >my friend claims they will not disagree. He says they will have the
> >same reading.
>
> [...]
>
> >He wants to see experimental verification.
> >Do you know of any?
>
> This would be pretty hard to do, you'd need a really big rocket engine and a
> very very sensitive clock.

I was wondering about the effect on particles (like Muons) with short
half lives, being accelerated in a linear accelerator. The effect of
*acceleration* on time should be measurable there. The only reference
that I've found, so far, only mentions relativistic speed as the
predominant fact affecting time.

> However the red shift between the bottom and top
> is something that obviously must happen when the system is viewed by
> an inertial observer. I don't know of any other way to describe this in the
> frame of the rocket other than as clocks running slower on the bottom.

Thanks for your comments,

Lawrence Foard

unread,
Apr 4, 2003, 2:06:15 PM4/4/03
to
In article <a42139e3.03040...@posting.google.com>,
Duwayne Anderson <duwa...@hotmail.com> wrote:
>ent...@farviolet.com (Lawrence Foard) wrote in message news:<b6hu4d$tvb$1...@farviolet.com>...
>> This would be pretty hard to do, you'd need a really big rocket engine and a
>> very very sensitive clock.
>
>I was wondering about the effect on particles (like Muons) with short
>half lives, being accelerated in a linear accelerator. The effect of
>*acceleration* on time should be measurable there. The only reference
>that I've found, so far, only mentions relativistic speed as the
>predominant fact affecting time.

This might be tricky. You can't necessarily make the muons stay in a form which
reflects the rocket ship. The rocket is rigid, the muon pulse however would be
free to rearrange itself. I suppose you could do it on a long rotating arm. But
this wouldn't be quite the same.

Joe Fischer

unread,
Apr 4, 2003, 3:21:21 PM4/4/03
to
Duwayne Anderson <duwa...@hotmail.com> wrote:
:Joe Fischer wrote:
:> Duwayne Anderson <duwa...@hotmail.com> wrote:
:> : Joe Fischer wrote:
:> :> Physics everywhere the same means all perfect
:> :> clocks keep perfect time.
:> :
:> : No, that's not what it means at all. There is nothing sacred about
:> : time that makes it invariant.
:>
:> I didn't say time was invariant, but there is
:> nothing that will make an ideal clock speed up or slow down.

If you are so sure, why are you having trouble
convincing your friend about the PoE?

: When compared to each other in different reference frames they do.

: It's an experimental fact.

Compared how, by radio (photon) signals?

:> Rate of time flow is tied to the length of measuring


:> rods, as is the speed of light.
:>
:> :> The facts about this are blurred by experiments
:> :> that try to compare clocks moving relative to each other
:> :> using radio (photon) signals, and that is difficult or
:> :> even an invalid endeavor.
:> :
:> : It's an experimental fact that clocks measure different amounts of
:> : time between events. For example, place an atomic clock on the
:> : ground, and another one in an airplane. Now have the airplane take
:> : off and fly around the world. When it lands, compare the clocks.
:> : They will have different readings. The difference between the
:> : readings is greater than can be attributed to clock error.
:>
:> But if this were true, it would be documented
:> with numerous experiments,
:
: There are.

Not that you can cite.

: During October 1971, a remarkable experiment was performed that


: checked both gravitational red shift and time dilation, by measuring
: directly their effects on traveling clocks. The actual experiment was
: coordinated by J. C. Hfele, then of the Washington University in St.
: Louis, and Richard Keating of the U.S. Naval Observatory, using
: cesium-beam atomic clocks. They flew their clocks on commercial
: aircraft during regularly scheduled flights (and because of government
: restrictions, could not even fly first class). The clocks were
: positioned against the front wall of the coach-class cabin to protect
: them from sudden motions. The eastward trip took place between
: October 4 and 7 and included 41 hours in flight, while the westward
: trip took place between October 13 and 17, and included 49 hours of
: flight. For the westward flight the predicted gain in the flying
: clock was 275 nanoseconds, of which two-thirds was due to
: gravitational blue shift; the observed gain was 273 nanoseconds. For
: the eastward flight, the time dilation was predicted to give a loss
: larger than the gain due to the gravitational blue shift, the net
: being a loss of 40 nanoseconds; the observed loss was 59 nanoseconds.
: These results agreed exactly with General Relativity, to within the
: experimental error of +/- 20 nanoseconds. The experimental error was
: attributed to inaccuracies in the flight data and intrinsic variations
: in the rates of cesium clocks.

If you would read the raw description of that
experiment you would find questionable material.

: Similar experiments were conducted by Bob Vessot and NASA in 1976 when


: they flew an atomic clock in low earth orbit and then compared it's
: time (after the clock was brought back to earth) with that of an
: identical clock kept at the NASA tracking station at Merritt Island.
: When the clocks were brought back together their relative time had
: shifted in exact agreement with the General Theory of Relativity. [See
: "Was Einstein Right? Putting General Relativity to the Test," by
: Clifford M. Will, Basic Books, 1986, pages 42-43; 54-57.]

I have had that book for years, and like it
very much, but "compared in different reference frames"
confuses the issue.
There is the issue of observation at a distance,
and a separate issue of clocks slowing or speeding up.

If a GPS clock could be brought back to Earth,
how much time do you think it would have lost or gained?

Joe Fischer


--
3

Duwayne Anderson

unread,
Apr 9, 2003, 1:31:09 PM4/9/03
to
Joe Fischer <grav...@shell1.iglou.com> wrote in message news:<3e8de...@news.iglou.com>...

> Duwayne Anderson <duwa...@hotmail.com> wrote:
> :Joe Fischer wrote:
> :> Duwayne Anderson <duwa...@hotmail.com> wrote:
> :> : Joe Fischer wrote:
> :> :> Physics everywhere the same means all perfect
> :> :> clocks keep perfect time.
> :> :
> :> : No, that's not what it means at all. There is nothing sacred about
> :> : time that makes it invariant.
> :>
> :> I didn't say time was invariant, but there is
> :> nothing that will make an ideal clock speed up or slow down.
>
> If you are so sure, why are you having trouble
> convincing your friend about the PoE?

Because he's made up his mind.

>
> : When compared to each other in different reference frames they do.
> : It's an experimental fact.
>
> Compared how, by radio (photon) signals?

Certainly by photon signals.

>
> :> Rate of time flow is tied to the length of measuring
> :> rods, as is the speed of light.
> :>
> :> :> The facts about this are blurred by experiments
> :> :> that try to compare clocks moving relative to each other
> :> :> using radio (photon) signals, and that is difficult or
> :> :> even an invalid endeavor.
> :> :
> :> : It's an experimental fact that clocks measure different amounts of
> :> : time between events. For example, place an atomic clock on the
> :> : ground, and another one in an airplane. Now have the airplane take
> :> : off and fly around the world. When it lands, compare the clocks.
> :> : They will have different readings. The difference between the
> :> : readings is greater than can be attributed to clock error.
> :>
> :> But if this were true, it would be documented
> :> with numerous experiments,
> :
> : There are.
>
> Not that you can cite.

See citation below.

>
> : During October 1971, a remarkable experiment was performed that
> : checked both gravitational red shift and time dilation, by measuring
> : directly their effects on traveling clocks. The actual experiment was
> : coordinated by J. C. Hfele, then of the Washington University in St.
> : Louis, and Richard Keating of the U.S. Naval Observatory, using
> : cesium-beam atomic clocks. They flew their clocks on commercial
> : aircraft during regularly scheduled flights (and because of government
> : restrictions, could not even fly first class). The clocks were
> : positioned against the front wall of the coach-class cabin to protect
> : them from sudden motions. The eastward trip took place between
> : October 4 and 7 and included 41 hours in flight, while the westward
> : trip took place between October 13 and 17, and included 49 hours of
> : flight. For the westward flight the predicted gain in the flying
> : clock was 275 nanoseconds, of which two-thirds was due to
> : gravitational blue shift; the observed gain was 273 nanoseconds. For
> : the eastward flight, the time dilation was predicted to give a loss
> : larger than the gain due to the gravitational blue shift, the net
> : being a loss of 40 nanoseconds; the observed loss was 59 nanoseconds.
> : These results agreed exactly with General Relativity, to within the
> : experimental error of +/- 20 nanoseconds. The experimental error was
> : attributed to inaccuracies in the flight data and intrinsic variations
> : in the rates of cesium clocks.
>
> If you would read the raw description of that
> experiment you would find questionable material.

I notice you failed to explain or support your assertion.

>
> : Similar experiments were conducted by Bob Vessot and NASA in 1976 when
> : they flew an atomic clock in low earth orbit and then compared it's
> : time (after the clock was brought back to earth) with that of an
> : identical clock kept at the NASA tracking station at Merritt Island.
> : When the clocks were brought back together their relative time had
> : shifted in exact agreement with the General Theory of Relativity. [See
> : "Was Einstein Right? Putting General Relativity to the Test," by
> : Clifford M. Will, Basic Books, 1986, pages 42-43; 54-57.]
>
> I have had that book for years, and like it
> very much, but "compared in different reference frames"
> confuses the issue.
> There is the issue of observation at a distance,
> and a separate issue of clocks slowing or speeding up.

Reference frames are *the* issue, and when clocks are separated,
travel in different reference frames, and return, they show different
time.

> If a GPS clock could be brought back to Earth,
> how much time do you think it would have lost or gained?

See the experiment I cited above, in which two atomic clocks were
tested. One stayed on the ground, and one went up in an airplane.

Duwayne Anderson

Joe Fischer

unread,
Apr 10, 2003, 3:29:40 PM4/10/03
to
Duwayne Anderson <duwa...@hotmail.com> wrote:
: Joe Fischer wrote:
:> : When compared to each other in different reference frames they do.
:> : It's an experimental fact.
:>
:> Compared how, by radio (photon) signals?
:
: Certainly by photon signals.

Photon signals do not show what clocks at
a distance read, and photon signals do not show
accurately what the rate of time flow as read
by the clock at a distance, and the emissions
timed to the clock at a distance do not show
accurately the emission frequency.

:> :> Rate of time flow is tied to the length of measuring

:> :> rods, as is the speed of light.
:> :>
:> :> :> The facts about this are blurred by experiments
:> :> :> that try to compare clocks moving relative to each other
:> :> :> using radio (photon) signals, and that is difficult or
:> :> :> even an invalid endeavor.
:> :> :
:> :> : It's an experimental fact that clocks measure different amounts of
:> :> : time between events. For example, place an atomic clock on the
:> :> : ground, and another one in an airplane. Now have the airplane take
:> :> : off and fly around the world. When it lands, compare the clocks.
:> :> : They will have different readings. The difference between the
:> :> : readings is greater than can be attributed to clock error.
:> :>
:> :> But if this were true, it would be documented
:> :> with numerous experiments,
:> :
:> : There are.
:>
:> Not that you can cite.
:
: See citation below.
:
:> : During October 1971, a remarkable experiment was performed that
:> : checked both gravitational red shift and time dilation, by measuring
:> : directly their effects on traveling clocks.

A very old experiment, which does not show
whether or not the clocks will read the same after
being co-located again.

:> If you would read the raw description of that


:> experiment you would find questionable material.
:
: I notice you failed to explain or support your assertion.

Any single experimental results are questionable
until supported by many repeated experiments.
And it is well known that the clocks suffered
some measure of lost time due to hard acceleration and
jerk on landing etc. Repeated experiments would
be able to remove these problems.

:> : Similar experiments were conducted by Bob Vessot and NASA in 1976 when


:> : they flew an atomic clock in low earth orbit and then compared it's
:> : time (after the clock was brought back to earth) with that of an
:> : identical clock kept at the NASA tracking station at Merritt Island.
:> : When the clocks were brought back together their relative time had
:> : shifted in exact agreement with the General Theory of Relativity. [See
:> : "Was Einstein Right? Putting General Relativity to the Test," by
:> : Clifford M. Will, Basic Books, 1986, pages 42-43; 54-57.]
:>
:> I have had that book for years, and like it
:> very much, but "compared in different reference frames"
:> confuses the issue.
:> There is the issue of observation at a distance,
:> and a separate issue of clocks slowing or speeding up.
:
: Reference frames are *the* issue, and when clocks are separated,
: travel in different reference frames, and return, they show different
: time.

Why? In low Earth orbit the major effect
would be because of relative velocity, not gravity,
and relative velocity without acceleration has no
supported or even purported effect on clocks.

:> If a GPS clock could be brought back to Earth,


:> how much time do you think it would have lost or gained?
:
: See the experiment I cited above, in which two atomic clocks were
: tested. One stayed on the ground, and one went up in an airplane.
: Duwayne Anderson

GPS is at 11,000 miles, which causes a substantial
gravity effect. Experiments need to try to focus
on either relative velocity or gravity.

With all the Space Shuttle flights and the long
term orbits of Skylab, Mir and the International Space
Station, why hasn't the issue been solidly nailed down
with many repeated experiments?

I think you are making assumptions about what
the effects the three types of relative motion cause,
and assumptions about whether it is the observer that
interprets those effects or if clocks really and truely
are affected.

I expect that it will become known that some
of the assumptions about individual experiments were
misinterpreted.

Joe Fischer

--
3

Tom Roberts

unread,
Apr 10, 2003, 4:45:50 PM4/10/03
to duwayne.r...@tek.com
Duwayne Anderson wrote:
> Given: The force required to keep every object separated in, Figure 1,
> at a constant distance from the floor is caused by one of
> the following:
>
> 1 Uniform Gravitational Field – Magnitude and direction of Force are

> independent of position in laboratory.
> 2 Linear acceleration – Magnitude and direction of Force are

> independent of position in laboratory.
> [...]
> Note: It is assumed that if the force is caused by acceleration, the
> velocities involved are significantly less than the speed of light.

This caveat is not required when one analyzes the situation correctly.


> Based on concepts from Einstein's Theories, it is understood that:

> • If the force is caused by Gravity, then the natural frequency of


> Clock B should increase if it is moved from level 1 to 2.

Right. (I don't have your picture, but from context I conclude level 2
is higher than level 1).

I'm ignoring your poor wording. This is not rally a change in
the "natural frequency" of the clock, but rather a change
in the COMPARISON of one clock to another.


> • If the force is caused by Linear acceleration, then after Clock B is


> moved to level 2, it will continue to be accelerated and move with the
> same velocity as Clock A.

Wrong. The box must maintiain its proper height, which requires that
top and bottom have different proper accelerations, and that implies
that the two clocks will have different rates (when compared via
electromagnetic signals between them). If you work it out in detail,
you'll find that this rate difference is EXACTLY the same as for your
case 1 in the uniform gravitational field. (If the box does not
maintain its proper height, and top and bottom have the same proper
acceleration, then it will be ripped apart; your description did not
mention that happening.)

To see this, look at it from an inertial frame in which the box is
initially at rest, but is accelerating upward. As the box accelerates,
Lorentz contraction makes it measure to be successively shorter over
time in this inertial frame, so at any given time (in this frame) the
upper clock has a lower velocity wrt this frame than does the lower
clock; so the upper clock must "tick faster" than the lower clock.

Such acceleration while maintaining proper length is known
as Born rigid motion. Note that for a solid object the
inter-molecular bonds will naturally behave this way when
accelerated by a push from behind or a pull from ahead
(as long as the push or pull is small compared to the bond
strengths, and after waiting for transients to decay away).

This is known as "Bell's paradox" (initially described as a pair of
accelerating spaceships connected by a string).


Tom Roberts tjro...@lucent.com

Duwayne Anderson

unread,
Apr 11, 2003, 3:13:56 PM4/11/03
to
Tom Roberts <tjro...@lucent.com> wrote in message news:<3E95D7FE...@lucent.com>...

Tom, thanks for your comments. I would like to clarrify that the
argument is not one that I'm advocating. Rather, it is one that has
been proposed to me by a relative. I've argued back using essentially
the same line of reasoning you have posted.

Duwayne Anderson

Duwayne Anderson

unread,
Apr 11, 2003, 3:20:59 PM4/11/03
to
Joe Fischer <grav...@shell1.iglou.com> wrote in message news:<3e95c...@news.iglou.com>...

> Duwayne Anderson <duwa...@hotmail.com> wrote:
> : Joe Fischer wrote:
> :> : When compared to each other in different reference frames they do.
> :> : It's an experimental fact.
> :>
> :> Compared how, by radio (photon) signals?
> :
> : Certainly by photon signals.
>
> Photon signals do not show what clocks at
> a distance read,

Sure they do. Otherwise you'd never know the time. You ALWAYS look
at clocks from a distance.

> and photon signals do not show
> accurately what the rate of time flow as read
> by the clock at a distance, and the emissions
> timed to the clock at a distance do not show
> accurately the emission frequency.

Sure they do. The most accurate clocks are based on atomic
resonances. So are photons. You can make a clock by stabilizing a
microwave laser and then counting the cycles.

>
> :> :> Rate of time flow is tied to the length of measuring
> :> :> rods, as is the speed of light.
> :> :>
> :> :> :> The facts about this are blurred by experiments
> :> :> :> that try to compare clocks moving relative to each other
> :> :> :> using radio (photon) signals, and that is difficult or
> :> :> :> even an invalid endeavor.
> :> :> :
> :> :> : It's an experimental fact that clocks measure different amounts of
> :> :> : time between events. For example, place an atomic clock on the
> :> :> : ground, and another one in an airplane. Now have the airplane take
> :> :> : off and fly around the world. When it lands, compare the clocks.
> :> :> : They will have different readings. The difference between the
> :> :> : readings is greater than can be attributed to clock error.
> :> :>
> :> :> But if this were true, it would be documented
> :> :> with numerous experiments,
> :> :
> :> : There are.
> :>
> :> Not that you can cite.
> :
> : See citation below.
> :
> :> : During October 1971, a remarkable experiment was performed that
> :> : checked both gravitational red shift and time dilation, by measuring
> :> : directly their effects on traveling clocks.
>
> A very old experiment, which does not show
> whether or not the clocks will read the same after
> being co-located again.

Droping two cannon balls from a tower is an "old experimnet," too.
And still as valid today as back then.

The clocks *were* brought back together, and they *DID* read
differently. So relative time is an experimental fact.

Joe Fischer

unread,
Apr 12, 2003, 12:40:08 PM4/12/03
to
Duwayne Anderson <duwa...@hotmail.com> wrote:
:Joe Fischer wrote:
:> Duwayne Anderson <duwa...@hotmail.com> wrote:
:> : Joe Fischer wrote:
:> :> : When compared to each other in different reference frames they do.
:> :> : It's an experimental fact.
:> :>
:> :> Compared how, by radio (photon) signals?
:> :
:> : Certainly by photon signals.
:>
:> Photon signals do not show what clocks at
:> a distance read,
:
: Sure they do. Otherwise you'd never know the time. You ALWAYS look
: at clocks from a distance.

Please take a physics course and an English course
and learn what "at a distance" means.

In all cases, not only the time of light travel,
but also relative velocity is important, with the vector
of the relative velocity requiring the use of trig.

:> and photon signals do not show

:> accurately what the rate of time flow as read
:> by the clock at a distance, and the emissions
:> timed to the clock at a distance do not show
:> accurately the emission frequency.
:
: Sure they do. The most accurate clocks are based on atomic
: resonances. So are photons. You can make a clock by stabilizing a
: microwave laser and then counting the cycles.

Crystal clocks may work on resonance, but atomic
clocks work by emitted frequency, which does not have
a regular cadence, but a random, though regular, pattern.

:> :> : During October 1971, a remarkable experiment was performed that


:> :> : checked both gravitational red shift and time dilation, by measuring
:> :> : directly their effects on traveling clocks.
:>
:> A very old experiment, which does not show
:> whether or not the clocks will read the same after
:> being co-located again.
:
: Droping two cannon balls from a tower is an "old experimnet," too.
: And still as valid today as back then.

Why would they drop two cannon balls? And
dropping a cannon ball and a marble is not a valid
experiment to test equal falling.
The only reason that a marble and a cannon
ball were expected to fall with different acceleration
was a wrong and antiquated perception that gravity
is an attractive force, and Newton's formulas actually
prove that view wrong.

A marble and a cannon ball cannot fall with
different accelerations, simply because they would
"pull" on the earth in the same direction even if
the moronic concept of attrictive force was valid.
And the mass of the Earth is so great,
it would be impossible to measure a difference even
if there was a difference, the measurement would
require 22 digit precision, and the best attained
so far is about 12 or 14 digits.

General Relativity completely removes the
concept of an attractive force acting at a distance,
and the concept of equal falling assumes a totally
different perspective, because General Relativity
considers the different attributes "gravitational
mass" and "inertial mass" to the _SAME_ attribute,
not merely two equal quantities.

: The clocks *were* brought back together, and they *DID* read


: differently. So relative time is an experimental fact.
: <snip to end>
: Duwayne Anderson

That has not been done repeatably or in the
the number of experiments that would have been possible
with all the many space flights.

Any use of photon signals to measure anything
requires using General Relativity to determine what
is actually being seen, without the distortions of
frequencu change and time of light travel.

Joe Fischer

--
3

Double-A

unread,
Apr 12, 2003, 7:08:44 PM4/12/03
to
Tom Roberts <tjro...@lucent.com> wrote in message news:<3E95D7FE...@lucent.com>...
> Duwayne Anderson wrote:
> > Given: The force required to keep every object separated in, Figure 1,
> > at a constant distance from the floor is caused by one of
> > the following:
> >
> > 1 Uniform Gravitational Field ? Magnitude and direction of Force are

> > independent of position in laboratory.
> > 2 Linear acceleration ? Magnitude and direction of Force are

> > independent of position in laboratory.
> > [...]
> > Note: It is assumed that if the force is caused by acceleration, the
> > velocities involved are significantly less than the speed of light.
>
> This caveat is not required when one analyzes the situation correctly.
>
>
> > Based on concepts from Einstein's Theories, it is understood that:
> > ? If the force is caused by Gravity, then the natural frequency of

> > Clock B should increase if it is moved from level 1 to 2.
>
> Right. (I don't have your picture, but from context I conclude level 2
> is higher than level 1).
>
> I'm ignoring your poor wording. This is not rally a change in
> the "natural frequency" of the clock, but rather a change
> in the COMPARISON of one clock to another.
>
>
> > ? If the force is caused by Linear acceleration, then after Clock B is


Does this mean that an accelerometer placed at the front of an
accelerating (9.8 m/sec^2) spaceship would display a lower
acceleration reading than an accelerometer placed at the rear of the
ship near the thrusters? Would the difference in the readings be
comparable to the difference that would be seen if the ship were
sitting upright on the ground in the earth's gravitational field?

Double-A

David

unread,
Apr 13, 2003, 5:47:13 PM4/13/03
to
doub...@hush.com (Double-A) wrote in message news:<79094630.03041...@posting.google.com>...

> Tom Roberts <tjro...@lucent.com> wrote in message news:<3E95D7FE...@lucent.com>...
> > Such acceleration while maintaining proper length is known
> > as Born rigid motion. Note that for a solid object the
> > inter-molecular bonds will naturally behave this way when
> > accelerated by a push from behind or a pull from ahead
> > (as long as the push or pull is small compared to the bond
> > strengths, and after waiting for transients to decay away).
> >
> > This is known as "Bell's paradox" (initially described as a pair of
> > accelerating spaceships connected by a string).
> >
> >
> > Tom Roberts tjro...@lucent.com
>
>
> Does this mean that an accelerometer placed at the front of an
> accelerating (9.8 m/sec^2) spaceship would display a lower
> acceleration reading than an accelerometer placed at the rear of the
> ship near the thrusters?

It does.

> Would the difference in the readings be
> comparable to the difference that would be seen if the ship were
> sitting upright on the ground in the earth's gravitational field?
>
> Double-A

Yes, it would be. The difference is that in the presence of the
earth's gravitational field the side walls are not accelerated by
gravity in exactly a parrallel downward direction, but have a small
component of the acceleration toward eachother.

Tom Roberts

unread,
Apr 13, 2003, 10:19:47 PM4/13/03
to
Double-A wrote:
> Tom Roberts <tjro...@lucent.com> wrote in message news:<3E95D7FE...@lucent.com>...
>>[...] The box must maintiain its proper height, which requires that
>>top and bottom have different proper accelerations, and that implies
>>that the two clocks will have different rates (when compared via
>>electromagnetic signals between them).
>
> Does this mean that an accelerometer placed at the front of an
> accelerating (9.8 m/sec^2) spaceship would display a lower
> acceleration reading than an accelerometer placed at the rear of the
> ship near the thrusters?

Yes of course. An accelerometer measures its own proper acceleration.


> Would the difference in the readings be
> comparable to the difference that would be seen if the ship were
> sitting upright on the ground in the earth's gravitational field?

Yes, as long as the inhomogeneity in the earth's field over the rocket
can be neglected (i.e. is below the measurement accuracy). This
difference is NOT due to inhomogeneities in the earth's field, it is due
to the fact that the top and bottom of the rocket are instantaneously at
rest in (slightly) different inertial frames. Note this difference is
VERY small....

I have not done the computations, and I do not know whether
the inhomogeneity in the earth's field is actually smaller
than this effect. For a 100 meter tall rocket the earth's
gravitational acceleration varies by about 31 parts per million,
and I suspect that may well be larger than this effect. I also
doubt that real accelerometers can be made with such accuracy.
(Pound and Rebka type experiments have such resolutions, but
they are sensitive to potential difference, not acceleration.)


Tom Roberts tjro...@lucent.com

Joe Fischer

unread,
Apr 15, 2003, 3:26:00 PM4/15/03
to
Double-A <doub...@hush.com> wrote:
: Does this mean that an accelerometer placed at the front of an

: accelerating (9.8 m/sec^2) spaceship would display a lower
: acceleration reading than an accelerometer placed at the rear of the
: ship near the thrusters?

Your question is not specific enough, it should
state if you mean in deep space far from other matter,
or in a gravitational "field".

At least one of the responses seems to answer
from the perspective of being in a gravitational "field".

An accelerometer by itself is not able to tell
if the spaceship is in a gravitational "field" or not,
it will read what the engine thrust would logically
suggest whether or not there is a gravitational "field"
present.
Comparison of readings of two accelerometers
is not meaningful, and has to be interpreted in respect
to observers and the model they are thinking of.

: Would the difference in the readings be


: comparable to the difference that would be seen if the ship were
: sitting upright on the ground in the earth's gravitational field?
: Double-A

This question seems to make assumptions and
then seems to expect the responses be made by those
making the same assumptions.

There is _NO_ gravitational "field", and
the only thing that thinking of a gravitational
"field" does in both of your questions is cause
confusion about what happens when the rocket
hovers slightly above the solid ground and the
shuts off the engines when it touches the ground.

If carefully done, the accelerometer
will read the same when the engines are thrusting
and when they are shut down.

Physics is logical, as is math, and
what remains to be determined is why the same
proper acceleration exists for the spaceship
not touching the ground with engines thrusting
and for the spaceship on the ground after the
engines are shut down.

Logic, in the absence of Newtonian bias,
should at least consider the possibility that
there is no difference!

Proper acceleration is proper acceleration,
regardless of where and when.

Joe Fischer

--
3

Duwayne Anderson

unread,
Apr 15, 2003, 6:47:07 PM4/15/03
to
Joe Fischer <grav...@shell1.iglou.com> wrote in message news:<3e984...@news.iglou.com>...
> Duwayne Anderson <duwa...@hotmail.com> wrote:
<snip>

> Please take a physics course and an English course
> and learn what "at a distance" means.

Feeling testy?

>
> In all cases, not only the time of light travel,
> but also relative velocity is important, with the vector
> of the relative velocity requiring the use of trig.

I didn't say otherwise. Why bring up strawmen?


> :> and photon signals do not show
> :> accurately what the rate of time flow as read
> :> by the clock at a distance, and the emissions
> :> timed to the clock at a distance do not show
> :> accurately the emission frequency.
> :
> : Sure they do. The most accurate clocks are based on atomic
> : resonances. So are photons. You can make a clock by stabilizing a
> : microwave laser and then counting the cycles.
>
> Crystal clocks may work on resonance, but atomic
> clocks work by emitted frequency, which does not have
> a regular cadence, but a random, though regular, pattern.

Atomic clocks work on atomic resonances.

> :> :> : During October 1971, a remarkable experiment was performed that
> :> :> : checked both gravitational red shift and time dilation, by measuring
> :> :> : directly their effects on traveling clocks.
> :>
> :> A very old experiment, which does not show
> :> whether or not the clocks will read the same after
> :> being co-located again.
> :
> : Droping two cannon balls from a tower is an "old experimnet," too.
> : And still as valid today as back then.
>
> Why would they drop two cannon balls?

You obviously missed the point, which is that just because an
experiment is "old" does not make it invalid.

<snip>


> : The clocks *were* brought back together, and they *DID* read
> : differently. So relative time is an experimental fact.
> : <snip to end>
> : Duwayne Anderson
>
> That has not been done repeatably or in the
> the number of experiments that would have been possible
> with all the many space flights.

The experiments were valid. The errors determined, and the results in
full agreement with GR.

> Any use of photon signals to measure anything
> requires using General Relativity to determine what
> is actually being seen, without the distortions of
> frequencu change and time of light travel.

In the experiment above, the clocks were compared. One went up. The
other stayed down. They disagreed in their measurment, by the exact
amount predicted by GR (to within experimental error).

Thus, time dilation is an experimental fact.

Duwayne Anderson

Double-A

unread,
Apr 16, 2003, 10:01:50 PM4/16/03
to
Joe Fischer <grav...@shell1.iglou.com> wrote in message news:<3e9c5...@news.iglou.com>...


Thank you for your reply. Your discussion on the effects in a rocket
thrusting near the ground was very interesting. But what I was trying
to clarify is a point about the equivalency principle of GR. We now
that an accelerometer can tell the difference between the apparent
gravitational acceleration on the ground floor of a tall building, and
that on the top floor. If one had a spacecraft of the same
proportions as the tall building, would one be able to tell by
accelerometer readings at the nose and tail ends of the ship, whether
one was accelerating through space far away from any gravitational
influence, or sitting upright on a non-rotating planet? Also assuming
these are the only two possibilities.

Double-A

dlzc@aol.com (formerly)

unread,
Apr 16, 2003, 10:31:35 PM4/16/03
to
Dear Double-A:

"Double-A" <doub...@hush.com> wrote in message
news:79094630.0304...@posting.google.com...
...


> Thank you for your reply. Your discussion on the effects in a rocket
> thrusting near the ground was very interesting. But what I was trying
> to clarify is a point about the equivalency principle of GR. We now
> that an accelerometer can tell the difference between the apparent
> gravitational acceleration on the ground floor of a tall building, and
> that on the top floor. If one had a spacecraft of the same
> proportions as the tall building, would one be able to tell by
> accelerometer readings at the nose and tail ends of the ship, whether
> one was accelerating through space far away from any gravitational
> influence, or sitting upright on a non-rotating planet? Also assuming
> these are the only two possibilities.

The equivalence principle is based on choosing a "box" small enough that
your instruments cannot tell the difference. By arbitrarily enlarging the
box, you can of course find a point where a variance can be detected.

The long ship accelerating in space would have uniform acceleration no
matter where you were inside... as long as its structure does not collapse.

You could also measure acceleration in a ship that was moving away from
Earth at a constant 1 cm/sec, and see the acceleration slowly get less and
less.

David A. Smith


Joe Fischer

unread,
Apr 16, 2003, 11:07:19 PM4/16/03
to
Double-A <doub...@hush.com> wrote:
: Thank you for your reply. Your discussion on the effects in a rocket

: thrusting near the ground was very interesting. But what I was trying
: to clarify is a point about the equivalency principle of GR.

The Einstein Principle of Equivalence is different
from the Newton/Galileo equivalence principle.

And the original Einstein Principle of equivalence
was simplistic in that it only related to a two dimensional
surface accelerating upward (like the surface of Earth or
the floor of a room).

: We [k]now


: that an accelerometer can tell the difference between the apparent
: gravitational acceleration on the ground floor of a tall building, and
: that on the top floor.

That is true if the mean density of the building
is the same or less than the mean density of surface
material of the Earth.
A gravimeter _is_ an accelerometer calibrated
to a very narrow range of readings.
At the top of a mountain the acceleration of
gravity is only about half the difference that would
be expected for the known density of the mountain.

: If one had a spacecraft of the same


: proportions as the tall building, would one be able to tell by
: accelerometer readings at the nose and tail ends of the ship, whether
: one was accelerating through space far away from any gravitational
: influence, or sitting upright on a non-rotating planet?

My opinion is that the density of the material
of the ship would matter, but past experiments assume
an "attraction", and that affects the construction of
the experiment.
What Einstein did was to extend the thought
experiment beyond the Newtonian assumptions, and
found that the original PoE and his calculations
from 1911 were wrong (because they only covered
the Newtonian part of the phenomenon.

: Also assuming


: these are the only two possibilities.
: Double-A

Why would you want to do that? Physics
has to go in any direction and to any distance
to find more and more conclusive answers.
Isn't it obvious that not enough is
known at present to make any final conclusions
about gravity?

I think the mathematicians are missing
the boat, in General Relativity the metric has
to be calculated at some point, and it can be
calculated at various times during a calculation,
and it should be possible to establish a trend
for the metric during the experiment.
If there is a consistent change in the
metric during the experiment, then it might be
possible to think about what could cause the
change.

Space-time is the mapping of events
to a space-time diagram, and since space is
just another word for distance between points,
then changes in the metric might be related
to something more physical than a space-time
diagram.

All experiments with accelerometers
must remove any Newtonian bias, else all that
is being done is doing the same experiment
with possibly better precision.
New experiments and new thinking
is needed.

Joe Fischer

--
3

Sebastian Pucilowski

unread,
May 14, 2003, 7:53:19 AM5/14/03
to
Good evening, sir's and miss's.

I was on the usual long train trip today, and I was pondering some
relativity (as you do!). I was trying to nut it out, but sadly I couldn't,
so I turn to this lively discussion group for some thoughts.

Its hypothetical, but that shouldn't matter, physics being physics
:).

Suppose we have three particles, X, Y and Z. Now, in the reference frame
of Y, I define X to be moving at speed c towards the "left", and Z to be
moving at speed c towards the "right".

Now, suppose we change the reference frame across to the particle Z.

My question, and source of confusion, is what on earth is happening to
particles X and Y? That is, what speed are they travelling, are they
making distance between them, and so on? From an engineering point of
view, X would be travelling at 2c, and Y at c. But clearly, this is an
impossible situation.

I spoke to a few collegues, and they suggested that its possible that both
are moving towards the "left" at c. I tried to bend this in with some
Lorentz transforms, but if that is so, then they aren't getting any closer
or further apart?

A very large hmm...

Your thoughts, kind sir's and miss's?


PS: I'm doing mechanical engineering and pure/applied mathematics at
Melbourne university, but I'll be dropping the engineering for the rigor
and delights of some delightful physics :)

--
Sebastian Pucilowski
The University of Melbourne
s.pucilowski over at unimelb.edu.au

Pmb

unread,
May 14, 2003, 8:45:48 AM5/14/03
to

"Sebastian Pucilowski" <sa...@athena.domain.local> wrote in message
news:Pine.LNX.4.44.030514...@athena.domain.local...

> Good evening, sir's and miss's.
>
> I was on the usual long train trip today, and I was pondering some
> relativity (as you do!). I was trying to nut it out, but sadly I couldn't,
> so I turn to this lively discussion group for some thoughts.
>
> Its hypothetical, but that shouldn't matter, physics being physics
> :).
>
> Suppose we have three particles, X, Y and Z. Now, in the reference frame
> of Y, I define X to be moving at speed c towards the "left", and Z to be
> moving at speed c towards the "right".

You mean like this

<-------X(-c) Y(at rest) Z(+c)--------->

For this to be true the particle must have zero proper mass. In which case
the speed of a particle proper mass is 'c' in all inertial frames of
referance.

If X is moving to the left and Z is moving to the right then the relative
speed of X with respect to Z is 2c. But that is not a problem since all
relativity says is that the speed of a single particle with respect to an
inertial frame of referance cannot be greater than 'c' (and equal to c if
the proper mass is zero). It does not say that the relative speed of one
particle with respect to another particle can't be greater than 'c'.

> Now, suppose we change the reference frame across to the particle Z.

This is the problem. You can't change to a frame of referance which is
moving at the speed of light. If you look at the Lorentz transformation
you'll notice that the Lorentz gamma factor becomes undefined when the speed
of the referance frame you're chaning to is moving at speed v = c

Pmb


Alfred Centauri

unread,
May 14, 2003, 8:46:15 AM5/14/03
to

"Sebastian Pucilowski" <sa...@athena.domain.local> wrote in message
news:Pine.LNX.4.44.030514...@athena.domain.local...
> Good evening, sir's and miss's.
>
> I was on the usual long train trip today, and I was pondering some
> relativity (as you do!). I was trying to nut it out, but sadly I couldn't,
> so I turn to this lively discussion group for some thoughts.
>
> Its hypothetical, but that shouldn't matter, physics being physics
> :).
>
> Suppose we have three particles, X, Y and Z. Now, in the reference frame
> of Y, I define X to be moving at speed c towards the "left", and Z to be
> moving at speed c towards the "right".
>
> Now, suppose we change the reference frame across to the particle Z.
>

Can't be done (according to SR). Since particle Z (and X) are moving at the
speed of light as observed from the reference frame at rest wrt particle Y,
particle Z (and X) will be observed to moving at the speed of light from
*all* inertial reference frames. In other words, there is no inertial frame
of reference in which particle Z (or X) is at rest.

Regards,

Alfred

Alfred Centauri

unread,
May 14, 2003, 8:46:15 AM5/14/03
to

"Sebastian Pucilowski" <sa...@athena.domain.local> wrote in message
news:Pine.LNX.4.44.030514...@athena.domain.local...
> Good evening, sir's and miss's.
>
> I was on the usual long train trip today, and I was pondering some
> relativity (as you do!). I was trying to nut it out, but sadly I couldn't,
> so I turn to this lively discussion group for some thoughts.
>
> Its hypothetical, but that shouldn't matter, physics being physics
> :).
>
> Suppose we have three particles, X, Y and Z. Now, in the reference frame
> of Y, I define X to be moving at speed c towards the "left", and Z to be
> moving at speed c towards the "right".
>
> Now, suppose we change the reference frame across to the particle Z.
>

Can't be done (according to SR). Since particle Z (and X) are moving at the

Hayek

unread,
May 14, 2003, 11:41:01 AM5/14/03
to

Sebastian Pucilowski wrote:

> Good evening, sir's and miss's.
>
> I was on the usual long train trip today, and I was pondering some
> relativity (as you do!). I was trying to nut it out, but sadly I couldn't,
> so I turn to this lively discussion group for some thoughts.
>
> Its hypothetical, but that shouldn't matter, physics being physics
> :).
>
> Suppose we have three particles, X, Y and Z. Now, in the reference frame
> of Y, I define X to be moving at speed c towards the "left", and Z to be
> moving at speed c towards the "right".
>
> Now, suppose we change the reference frame across to the particle Z.
>
> My question, and source of confusion, is what on earth is happening to
> particles X and Y? That is, what speed are they travelling, are they
> making distance between them, and so on? From an engineering point of
> view, X would be travelling at 2c, and Y at c. But clearly, this is an
> impossible situation.
>
> I spoke to a few collegues, and they suggested that its possible that both
> are moving towards the "left" at c. I tried to bend this in with some
> Lorentz transforms, but if that is so, then they aren't getting any closer
> or further apart?
>
> A very large hmm...
>
> Your thoughts, kind sir's and miss's?


Let us make it a bit more intresting.
Let's have speeds of 0.6c in each direction.

X sends a light beam to Z, will it ever arrive ?

Yes, because it will travel at c wrt to Y, and easily
reach or overtake Z.

Works with .99 c in each direction also.

Hayek.

--
The small particles wave at
the big stars and get noticed.
:-)

kenseto

unread,
May 14, 2003, 12:56:04 PM5/14/03
to

"Sebastian Pucilowski" <sa...@athena.domain.local> wrote in message
news:Pine.LNX.4.44.030514...@athena.domain.local...
> Good evening, sir's and miss's.
>
> I was on the usual long train trip today, and I was pondering some
> relativity (as you do!). I was trying to nut it out, but sadly I couldn't,
> so I turn to this lively discussion group for some thoughts.
>
> Its hypothetical, but that shouldn't matter, physics being physics
> :).
>
> Suppose we have three particles, X, Y and Z. Now, in the reference frame
> of Y, I define X to be moving at speed c towards the "left", and Z to be
> moving at speed c towards the "right".
>
> Now, suppose we change the reference frame across to the particle Z.
>
> My question, and source of confusion, is what on earth is happening to
> particles X and Y? That is, what speed are they travelling, are they
> making distance between them, and so on? From an engineering point of
> view, X would be travelling at 2c, and Y at c. But clearly, this is an
> impossible situation.
>
> I spoke to a few collegues, and they suggested that its possible that both
> are moving towards the "left" at c. I tried to bend this in with some
> Lorentz transforms, but if that is so, then they aren't getting any closer
> or further apart?
>
> A very large hmm...
>
> Your thoughts, kind sir's and miss's?

I posted a similar question in the thread "SR Questions". I suggest that you
take a look at the discussion there. The initial post is as follows:

We have three observers A, B and C in a straight line.
A----.9c----->B <------.9c------C
1. According to B, A is moving toward him at .9c
2. According to B, C is moving toward him at .9c.
3. According to B, A and C are approaching each other at 1.8c.
4. According to A, C is moving toward him at .994c.
5. According to C, A is moving toward him at .994c.

Question:
Why is B can see A and C approaching at each other at
1.8 c and yet A and C can only measure that they are
approaching each other at .994 c?

The solution:
1. A and C are indeed approaching each other at 1.8c
2. A's measurement of C's approaching speed of .994c is apparent and it is
due to that light is being transmitted by a medium at a max speed of c.
That's why A can only detect C's approaching speed to be less than c. A's
measurement can be corrected and making it equal to 1.8 c as follows:
Corrected approaching speed of C= .994c(Fac/Faa)
Where Fac=frequency of a standard light source in C's frame as measured
by A
Faa=frequency of an identical light source in A's frame as
measured by A

Hope this helps.

Ken Seto


Dirk Van de moortel

unread,
May 15, 2003, 11:51:15 AM5/15/03
to

"kenseto" <ken...@erinet.com> wrote in message news:vc4t5l5...@corp.supernews.com...

It will surely help Sebastian in his struggle to find out what an
idiot you are.
http://users.pandora.be/vdmoortel/dirk/Physics/Fumbles/HTHelps.html
Title: "Hope this helps".

Dirk Vdm


0 new messages